Yahoo France Recherche Web

Résultats de recherche

  1. Get the range of the required distribution, in this case, max(X, Y) Find the CDF of this distribution as a function of the known distributions Find the PDF of the distribution by differentiating the CDF

  2. $\begingroup$ I prefer $\max\{f(x_1,\ldots,f(x_n)\}$ with curly braces and no parentheses. In this instance, the parentheses don't actually help, and the curly braces remind you that the thing whose maximum is sought is a set rather than a tuple. $\endgroup$

  3. Stack Exchange Network. Stack Exchange network consists of 183 Q&A communities including Stack Overflow, the largest, most trusted online community for developers to learn, share their knowledge, and build their careers.

  4. Rewriting this, E[Z] ≤ logn t + tσ2 2. Now, set t = √2logn σ to get. E[Z] ≤ σ√2logn. The reason Sivaraman set t = \sqrt {2\log {n}}/\sigma is because that is the point at which the upper bound is at a minimum. You can see this by taking the derivative of the bound with respect to t and setting it to zero.

  5. Stack Exchange Network. Stack Exchange network consists of 183 Q&A communities including Stack Overflow, the largest, most trusted online community for developers to learn, share their knowledge, and build their careers.

  6. 6 juin 2016 · Stack Exchange Network. Stack Exchange network consists of 183 Q&A communities including Stack Overflow, the largest, most trusted online community for developers to learn, share their knowledge, and build their careers.

  7. 11 mai 2020 · But let's take x = 2, then (1 - 2) ^ 2 will be (-1) ^2 which is nothing but 1 and according to op's max function, 1 should be returned. But since you gave the condition of x >= 1, we always return 0 even when x is something like 2. I think in comments what Andre Holzner said is correct.

  8. 21 août 2011 · M (x) is a function. Taking the maximal number amongst the parameters. max {x1, x2} = {x1, if x1> x2 x2, otherwise. You can define like that the maximum of any finitely many elements. When the parameters are an infinite set of values, then it is implied that one of them is maximal (namely that there is a greatest one, unlike the set {− 1 n ...

  9. iPhone XS Max 的屏幕尺寸达到了惊人的 6.5 英寸,屏幕分辨率为 2688 × 1242 像素,像素密度为 458 ppi,屏幕精度跟 iPhone XS 保持一致,显示面积则比 iPhone 8 Plus 要大 21%,视觉冲击力自然也要强不少. 不过,考虑到 iPhone XS Max 的手势交互都集中在机身的下半段,长时间握 ...

  10. $\begingroup$ @sleevechen for (1) we can say it because $\left \| A \right \|$ is the maximum of all the elements in the set, and it was just shown that each element is $\leq $ the largest singular value times $\left \| x \right \| = 1$.

  1. Recherches associées